Board logo

標題: 104板橋高中 [打印本頁]

作者: rueichi    時間: 2015-5-23 13:26     標題: 104板橋高中

如檔

附件: 104板橋高中.pdf (2015-5-24 08:20, 329.34 KB) / 該附件被下載次數 10602
https://math.pro/db/attachment.php?aid=2873&k=0f18c204ce2d395cd5c00f061f0bf598&t=1713567485
作者: jkliopnm    時間: 2015-5-24 01:21

為什麼第一部分的填充2我一直算出162/25....
作者: thepiano    時間: 2015-5-24 08:44     標題: 回復 2# jkliopnm 的帖子

2.
設\(O\)為坐標平面的原點,若過點\(\displaystyle P\left(\frac{6}{5},\frac{12}{5}\right)\)的直線分別與\(x\)軸,\(y\)軸的正向交於\(A,B\)兩點,則當\(\Delta OAB\)周長為最小值時,\(\Delta OAB\)的面積為   
[提示]
當 A(3,0),B(0,4) 時,△OAB 有最小周長 12,此時 △OAB = 6
作者: EZWrookie    時間: 2015-5-24 11:00

想請教3.7.9.13.15
謝謝版上的老師們。
作者: tsusy    時間: 2015-5-24 13:23     標題: 回復 4# EZWrookie 的帖子

(3)
空間中兩直線\(L_1\):\(\displaystyle \frac{x-6}{-2}=\frac{y+5}{2}=\frac{z-5}{-1}\)與\(L_2\):\(\displaystyle \frac{x+2}{-4}=\frac{y-6}{7}=\frac{z-7}{4}\)的其中一條分角線方程式為\(\displaystyle \frac{x-6}{4}=\frac{y-b}{2}=\frac{z-c}{d}\),求\(b+c+d=\)   
[提示]
找交點,找分角線方向

(9)
設\(a,b,c\)為實數,二次方程式\(ax^2+bx+c=0\)的二根為\(\alpha,\beta\),其中\(-1\le \alpha \le 0\),\(1\le \beta \le 2\),若\(2a+b+c=4\),且\(a\ge 2\ge b\ge -8\),則\(a+3b+2c\)的最小值為   
[解答]
由 \( a>0 \) 知函數 \( f(x) = ax^2+bx+c \) 之圖形為開口向上的拋物線

又 \( f(x) = 0 \) 之兩根 \( \alpha, \beta \),故 \( f(-1) \geq 0, f(0) \leq 0, f(1) \leq 0, f(2) \geq 0 \)。

故 \( a,b \) 滿足  \( f(-1) \geq 0, f(0) \leq 0, f(1) \leq 0, f(2) \geq 0 \) 及 \( a\geq2 \), \( 2\geq b \geq -8 \)

以 \( c = 4 -2a -b \) 替換,可得一線性規劃問題(變數為 \( a,b \) ),以頂點法可找到最小值
作者: tuhunger    時間: 2015-5-24 13:43     標題: 第3題

空間中兩直線\(L_1\):\(\displaystyle \frac{x-6}{-2}=\frac{y+5}{2}=\frac{z-5}{-1}\)與\(L_2\):\(\displaystyle \frac{x+2}{-4}=\frac{y-6}{7}=\frac{z-7}{4}\)的其中一條分角線方程式為\(\displaystyle \frac{x-6}{4}=\frac{y-b}{2}=\frac{z-c}{d}\),求\(b+c+d=\)   

圖片附件: 3.png (2015-5-24 13:43, 17.79 KB) / 該附件被下載次數 6844
https://math.pro/db/attachment.php?aid=2876&k=6054e58a039cbd72ccb6bd923919c1a8&t=1713567485


作者: tuhunger    時間: 2015-5-24 14:10     標題: 第13題

若數列\(\langle\;a_n \rangle\;\)滿足\(\displaystyle (1+x+x^2)^{2015}=1+\sum_{k=1}^{4030}a_kx^k\),則\(a_1+a_5+a_9+\ldots+a_{4029}=\)   

圖片附件: 13.png (2015-5-24 14:10, 16.05 KB) / 該附件被下載次數 7078
https://math.pro/db/attachment.php?aid=2878&k=3429a25e3d1a1281082b163912379189&t=1713567485


作者: tuhunger    時間: 2015-5-24 16:03     標題: 第8,10,11題

8.
整係數三次函數\(f(x)=x^3+ax^2+bx+c\),已知\(\displaystyle \lim_{x\to 1}\frac{f(x)}{x^5-1}=2\),且\(a>b>0\),則數對\((a,b,c)=\)   

10.
設對任意實數\(x,y\),函數\(f(x)\)恆滿足\(f(x+y)=f(x)+f(y)+2xy\),且導數\(f'(0)=3\),則導函數\(f'(x)=\)   

11.
化簡\(\displaystyle \prod_{n=2}^{24}\frac{n^3-2n^2+2n-1}{n^3+2n^2+2n+1}=\)   

第8,10,11題 如圖檔

圖片附件: 8+10+11.png (2015-5-24 16:03, 22.43 KB) / 該附件被下載次數 7054
https://math.pro/db/attachment.php?aid=2879&k=a74fc28dc47955f407f7e125b15cb837&t=1713567485


作者: jyi    時間: 2015-5-24 16:07     標題: 回復 3# thepiano 的帖子

請問第二題詳細算式!謝謝!
作者: thepiano    時間: 2015-5-24 17:15     標題: 回復 10# jyi 的帖子

第2題
設\(O\)為坐標平面的原點,若過點\(\displaystyle P\left(\frac{6}{5},\frac{12}{5}\right)\)的直線分別與\(x\)軸,\(y\)軸的正向交於\(A,B\)兩點,則當\(\Delta OAB\)周長為最小值時,\(\Delta OAB\)的面積為   
[解答]
作PC垂直OA於C,PD垂直OB於D
令\(\angle BAO=\theta \)
\(\begin{align}
  & PC=\frac{12}{5},PD=\frac{6}{5} \\
& AC+AP=\frac{12}{5}\times \left( \frac{1}{\tan \theta }+\frac{1}{\sin \theta } \right) \\
& BD+BP=\frac{6}{5}\times \left( \tan \theta +\frac{1}{\cos \theta } \right) \\
\end{align}\)
周長\(=\frac{12}{5}\times \left( \frac{1}{\tan \theta }+\frac{1}{\sin \theta } \right)+\frac{6}{5}\times \left( \tan \theta +\frac{1}{\cos \theta } \right)+\frac{18}{5}\)
令\(\tan \frac{\theta }{2}=t\ \left( 0<t<1 \right)\)
\(\begin{align}
  & \frac{12}{5}\times \left( \frac{1}{\tan \theta }+\frac{1}{\sin \theta } \right)+\frac{6}{5}\times \left( \tan \theta +\frac{1}{\cos \theta } \right)+\frac{18}{5} \\
& =\frac{12}{5}\times \left( \frac{1-{{t}^{2}}}{2t}+\frac{1+{{t}^{2}}}{2t} \right)+\frac{6}{5}\times \left( \frac{2t}{1-{{t}^{2}}}+\frac{1+{{t}^{2}}}{1-{{t}^{2}}} \right)+\frac{18}{5} \\
& =\frac{12}{5}\times \left( 1+\frac{1-t}{t} \right)+\frac{6}{5}\times \left( 1+\frac{2t}{1-t} \right)+\frac{18}{5} \\
& =\frac{12}{5}\left( \frac{1-t}{t}+\frac{t}{1-t} \right)+\frac{36}{5} \\
& \ge \frac{24}{5}+\frac{36}{5} \\
& =12 \\
\end{align}\)
等號成立於\(t=\frac{1}{2}\)
此時\(OA=3,OB=4,\Delta OAB=6\)
作者: tuhunger    時間: 2015-5-24 17:52     標題: 第2題 另解

設\(O\)為坐標平面的原點,若過點\(\displaystyle P\left(\frac{6}{5},\frac{12}{5}\right)\)的直線分別與\(x\)軸,\(y\)軸的正向交於\(A,B\)兩點,則當\(\Delta OAB\)周長為最小值時,\(\Delta OAB\)的面積為   

圖片附件: 2.png (2015-5-24 18:35, 40.91 KB) / 該附件被下載次數 4541
https://math.pro/db/attachment.php?aid=2884&k=5545463f6f73720c7c682d2815182471&t=1713567485


作者: farmer    時間: 2015-5-24 20:17     標題: 回復 10# thepiano 的帖子

太佩服了。
這個問題擴展到一般的情況,
也就是\(P\)點坐標改為\((a,b)\),
不知有沒有通解表示法?
作者: farmer    時間: 2015-5-24 20:33     標題: 回復 11# tuhunger 的帖子

請問第一行周長=2r 是怎麼來的?
能否詳細說明您的圖形畫法?
作者: farmer    時間: 2015-5-24 21:00     標題: 回復 1# rueichi 的帖子

看來這是一份算得很過癮但是會邊算邊罵的題目,選題真的是關鍵了,好幾題會讓人走火入魔的題目,跳過才是上策。

7.
\(k\)為整數,若\(x^3+12x^2+(36+2k)x+280+12k=0\)有三個整數根,試求\(k=\)   
[解答]
第7題也是其中一題吧,
f(x)=x^3+12x^2+(36+2k)x+280+12k
設 f(x)=(x-a)(x-b)(x-c),其中a,b,c皆為整數,
則a+b+c=-12,ab+bc+ca=36+2k,abc=-280-12k
考慮f(-6)=(-6-a)(-6-b)(-6-c)=280,令a1=-6-a,b1=-6-b,c1=-6-c,則
(a1)(b1)(c1)=280.....................................(1)式
a1+b1+c1=-18-(a+b+c)=-6......................(2)式
(a1)(b1)+(b1)(c1)+(c1)(a1)=2k..........(3)式
由第(2)式可知a1,b1,c1為三偶或兩奇一偶,再由第(3)式知兩奇一偶不合,
因此為三偶,可設a1=2*(a2),b1=2*(b2),c1=2*(c2),
代入(1)式及(2)式可得
(a2)(b2)(c2)=35,
a2+b2+c2=-3,
可知a2、b2、c2之解為-1、-7、5,
依序帶回可得k= -66
作者: tsusy    時間: 2015-5-24 21:21     標題: 回復 13# farmer 的帖子

第2題.
設\(O\)為坐標平面的原點,若過點\(\displaystyle P\left(\frac{6}{5},\frac{12}{5}\right)\)的直線分別與\(x\)軸,\(y\)軸的正向交於\(A,B\)兩點,則當\(\Delta OAB\)周長為最小值時,\(\Delta OAB\)的面積為   
[提示]
腦補細節,或許 tuhunger 老師原本的想法更妙

先取 \( O' \) 滿足 \( \overline{O'P} = O' \) 到 \( x, y \) 軸的距離,如 #11 樓的計算得 \( O'(6,6) \)

做圓 \( O' \) 與 \( L \) 相切,因 \( \overline{O'P} = 6 \),故此圓之半徑 \( \leq 6 \),此圓落在第一象限之內或與坐標軸相切。

過 \( A, B \) 分別對圓 \( O' \) 做另一條異於 \( L \) 之切線,分別切圓 \( O' \) 於 \( R, S \)

則有 \( \overline{AR} + \overline{BS} = \overline{AB} \) (切線段長相等)

故\( \triangle OAB \) 之周長 \( = \overline{OA} + \overline{AR} + \overline{OB} + \overline{BS} \)

令 \( r \) 為圓 \( O' \) 之半徑 \( (\alpha,0) \) 為 \( A \) 之坐標,則 \( \overline{AR} = \sqrt{ 6^2 + (6-\alpha)^2 - r^2} \geq |6-\alpha| \)

因此 \( \overline{OA} + \overline{AR} \geq \alpha + |6-\alpha| \geq 6 \)

同理 \( \overline{OB} + \overline{BS} \geq 6 \)

綜合兩不等式有 \( \triangle OAB \) 之周長 \( \geq 12 \) 且當 \( L \) 與圓 \( O' \) 相切於 \( P \) 點時等號成立。
作者: farmer    時間: 2015-5-24 21:26

14.
\(x,y,z\)均為正實數,若滿足\(\cases{\displaystyle x\le \frac{z^2}{4+z^2}\cr \frac{y}{2}\le \frac{4x^2}{1+4x^2}\cr \frac{z}{4}\le \frac{y^2}{1+y^2}}\),試求所有可能的\(x=\)   
[解答]
第14題這種題目大概都要朝某個不等式成立的情況去想,
這題是要考慮算幾不等式,
將三個不等式右邊的分母使用算幾不等式,可得
x<=z/4,
y/2<=x,
z/4<=y/2,
可知三個算幾不等式皆須成立,
因此x=1/2,y=1,z=2
作者: thepiano    時間: 2015-5-24 21:45     標題: 回復 12# farmer 的帖子

\(\begin{align}
  & b\times \left( \frac{1-{{t}^{2}}}{2t}+\frac{1+{{t}^{2}}}{2t} \right)+a\times \left( \frac{2t}{1-{{t}^{2}}}+\frac{1+{{t}^{2}}}{1-{{t}^{2}}} \right)+a+b \\
& =b\times \left( 1+\frac{1-t}{t} \right)+a\times \left( 1+\frac{2t}{1-t} \right)+a+b \\
& =\frac{b\left( 1-t \right)}{t}+\frac{2at}{1-t}+2a+2b \\
& \ge 2\sqrt{2ab}+2a+2b \\
\end{align}\)
周長的最小值為\(2\sqrt{2ab}+2a+2b\)
(1)當\(2a=b\)時,等號成立於\(t=\frac{1}{2}\),此時面積為\(\frac{1}{2}\left( a+\frac{3}{4}b \right)\left( b+\frac{4}{3}a \right)=ab+\frac{2}{3}{{a}^{2}}+\frac{3}{8}{{b}^{2}}\)
(2)當\(2a>b\)時,等號成立於\(t=\frac{-b+\sqrt{2ab}}{2a-b}\),此時面積小弟不想算,留給有耐心的人,不會是\(ab+\frac{2}{3}{{a}^{2}}+\frac{3}{8}{{b}^{2}}\)就是了
(3)當\(2a<b\)時,等號成立於\(t=\frac{-b-\sqrt{2ab}}{2a-b}\)
作者: idontnow90    時間: 2015-5-24 23:07

想請教100桃園新進聯招的題目..也可以仿用#10的作法嗎?謝謝
題目如下:\(f(x+y)=f(x)+f(y)+x^2y+y^2x\),已知\( \displaystyle \lim_{x \to 0} \frac{f(x)}{x}=1 \),則\(f'(x)=\)?
ANS:\(1+x^2\)
作者: tuhunger    時間: 2015-5-24 23:37     標題: 回復 18# idontnow90 的帖子

應該可,但你題目是不是打錯了?
我覺得題目應該是\(f(x+y)=f(x)+f(y)+(x^2)y+(y^2)x…\)
若是這樣的話,答案是沒錯的

解法:對\(y\)微=>\(f'(x+y)=0+f'(y)+x^2+2xy\)
\(y=0\)代入,得\(f'(x)=f'(0)+x^2+0=1+x^2\)
作者: idontnow90    時間: 2015-5-24 23:44     標題: 回復 19# tuhunger 的帖子

你是對的..我忘了括號~thx
用這個方法我卡住哩..可以寫一下詳解嗎~謝謝~
作者: farmer    時間: 2015-5-24 23:56     標題: 回復 17# thepiano 的帖子

太棒了,thank you.
作者: tuhunger    時間: 2015-5-25 01:48     標題: 第2題 另解 (想法)

感謝寸絲老師給一個更嚴謹,更清楚的解釋方式...
我的想法大概是:
首先作與兩軸相切的圓O'...且圓O'與過P點的直線相切...
這樣的話,我們所求的Δ周長=2r
換言之,題目就可以改寫成:圓與兩軸和過P的直線相切,求此圓的直徑最小值?

經過幾次不嚴謹的操作,會發現當P點恰為切點時,此時的圓O'最小(why?)

設P為切點的直線為L,  且過P的割線為L1...
理由1 :"用眼睛"看會發現若圓O'要與L1相切...圓O'會變大(覺得用眼睛看不準,可參考理由2)
理由2 : 假設與L1相切的圓O'才是最小圓, 那麼作L2//L1,且L2切與P為切點那個圓O'
很明顯的,[與P為切點那個圓O' ]小於[與L1相切的圓O'], 原假設矛盾

圖片附件: 2.png (2015-5-25 01:48, 54.92 KB) / 該附件被下載次數 4825
https://math.pro/db/attachment.php?aid=2885&k=b25c7e9d9ad72edf1e04684d1c5e5af9&t=1713567485


作者: windin0420    時間: 2015-5-25 13:26

想請問一下

13和17

謝謝
作者: thepiano    時間: 2015-5-25 13:43     標題: 回復 23# windin0420 的帖子

第 13 題
前面有

第 17 題
考慮 mod 8
作者: windin0420    時間: 2015-5-25 14:07     標題: 回復 24# thepiano 的帖子

想通了!

謝謝

補充想通後的解法 真得很感謝thepiano大

第17題
17.
給定一個正整數\(N\)定義\(\displaystyle f(N;x)=\sum_{i=0}^n a_i x^i\),
其中\(a_0\)為\(N\)的個位數字,\(a_1\)為\(N\)的十位數字...,\(a_n\)為\(N\)的最高位數
例如:\(f(3456;x)=6+5x+4x^2+3x^3\),而\(f(3456;1)=6+5\times 1+4\times 1^2+3\times 1^3=18\)
若\(M=12345678910111213\ldots \ldots20142015\)
令\(b_1=f(M;2)\),\(b_{j+1}=f(b_j;2)\)其中\(j=1,2,3,4,\ldots\),試求\(\displaystyle \lim_{j \to \infty}b_j=\)   
[解答]
先舉個簡單例子 如f(abcd;2)

f(abcd;2)=d+c*2+b*2^2+a*2^3

原本abcd可表示成 d+c*10+b*10^2+a*10^3

則 abcd-f(abcd;2) 會是8的倍數 即 abcd=f(abcd;2) (mod 8)

因此 M=b(1)=b(2)=....=b(n) (mod 8)

當n夠大時 b(n)會是個位數

M (mod 8) = 7

7+8=15 & 7-8=-1 非個位數 所以只有一解7
作者: tuhunger    時間: 2015-5-25 23:51     標題: 第12題

12.
對於所有整數\(m,n\)定義\(\left(\matrix{n \cr m}\right)=\cases{\displaystyle \frac{n!}{m!(n-m)!},if n\ge m\ge 0\cr 0    ,otherwise}\)
及數列\(\langle\;a_n \rangle\;\)滿足\(\displaystyle a_n=\sum_{k=1}^{\infty}\left(\matrix{k \cr n-k}\right),n \in N\),則\(a_{17}\)的值為   
[解答]
除了暴力破解法外, 提供一個小暴力方式

圖片附件: 12.png (2015-5-25 23:51, 5.32 KB) / 該附件被下載次數 4797
https://math.pro/db/attachment.php?aid=2891&k=cad334aab18ae8234e36c886601f35b5&t=1713567485


作者: farmer    時間: 2015-5-26 00:19     標題: 回復 22# tuhunger 的帖子

很好的想法,thank you
作者: g112    時間: 2015-5-28 01:17

引用:
原帖由 tuhunger 於 2015-5-25 11:51 PM 發表
除了暴力破解法外, 提供一個小暴力方式
能請教一下為什麼可以看成是費式數列??
作者: tuhunger    時間: 2015-5-28 16:30     標題: 回復 28# g112 的帖子

http://episte.math.ntu.edu.tw/articles/sm/sm_05_07_1/page3.html
作者: tuhunger    時間: 2015-5-28 23:17     標題: 第6題 幾合解法

6.
若\(k\)為整數,且\(\displaystyle x\ne \frac{\pi}{2}+k\pi\),則函數\(\displaystyle f(x)=\frac{2tanx}{1+2secx}\)的最大值為   
[解答]
小弟提供一個幾合的解法供各位參考....(代數方式可用"正餘弦疊合"試試看)

圖片附件: 6.png (2015-5-28 23:17, 25.59 KB) / 該附件被下載次數 5089
https://math.pro/db/attachment.php?aid=2899&k=a90969f02a4dbba4ee94b582365f07ab&t=1713567485


作者: 阿光    時間: 2015-5-29 13:50

請教9,15,16題,謝謝
作者: thepiano    時間: 2015-5-29 14:15     標題: 回復 31# 阿光 的帖子

參考一下
http://www.shiner.idv.tw/teachers/viewtopic.php?f=53&t=3862
作者: tuhunger    時間: 2015-5-29 16:49     標題: 第16題

16.
數列\(\langle\;a_n\rangle\;\)、\(\langle\;b_n\rangle\;\)滿足\(a_1=-1\),\(b_1=1\),\(a_{n+1}=6a_n-6b_n\),\(b_{n+1}=2a_n-b_n\),請寫出\(b_{n+1}\)的一般式為   
[解答]
若題目只要求第\(n\)項, 可先算出特徵值,  就可以把 第\(n\)項的形式假設出來...

圖片附件: 16.png (2015-5-29 16:49, 23.54 KB) / 該附件被下載次數 5544
https://math.pro/db/attachment.php?aid=2900&k=217472c38c21f35963c81fec33b6da3c&t=1713567485


作者: anyway13    時間: 2021-11-26 00:52     標題: 請教第4題

已知\(a,b,c>0\)且\(log_{0.5}(a+b+c)+log_{0.5}a+log_{0.5}b+log_{0.5}c=0\),試求\(log_{0.5}(a+b)+log_{0.5}(a+c)\)最大值為   

板上老師好

請問第四題,知道條件abc(a+b+c)=1,欲求log(底0.5)(a+b)(a+c)最大  (找(a+b)(a+c)最小)

下面自己想法

看題b,c對稱所以就直接猜b=c去做了

令f(a,b)=a^2+2ab+b^2, g(a,b)=(ab)^2+2ab^3-1,利用lagrange condition倒三角形f=(浪達)倒三角形g

得到  (a+b)^3=2(ab^2+b^3)...(*)  接下來考慮(ab^2+b^3)  /2  大於等於根號(ab^2*b^3) 在等號成立時  (a=b,有可能達到最小)

猜a=b帶入(*),b=0然後  b>0(題意) 就卡住了
作者: thepiano    時間: 2021-11-26 06:53     標題: 回復 34# anyway13 的帖子

第 4 題
參考一下小弟的做法
http://www.shiner.idv.tw/teachers/download/file.php?id=2406
作者: anyway13    時間: 2021-11-27 03:27     標題: 回復 35# thepiano 的帖子

鋼琴老師再次賜教     十分感謝




歡迎光臨 Math Pro 數學補給站 (https://math.pro/db/) 論壇程式使用 Discuz! 6.1.0